Вы находитесь на странице: 1из 38

Criminal Law Levy II 2011 A.

Sources of Criminal Law:


a) Common Law Jurisdictions: The judge continues to play an important role in shaping criminal law. Judges definitively interpret the meaning of criminal statutes and occasionally go beyond the statutes and announce new rules of their own. a. Under common law, if the same act was both a felony and a misdemeanor, the misdemeanor merged into the felony and there could be no conviction for the misdemeanor. Under modern law, there is no doctrine of merger in the United States. b) Model Penal Code (MPC): In MPC jurisdictions, the court relies on the statutes and their language to determine what the law is. a. Functions as a guide for Legislators, but does not represent the law in any jurisdiction unless the jurisdiction adopts its provisions. b. Most have adopted the MPC, or at least substantial portions of it.

B.

Introduction and the Purposes of Punishment


a) Purposes of Punishment a. Deterrence General (deter others) or specific (deters offender from committing same crime); must punish criminals to stop others from doing the same act. Based on the premise that criminals weigh advantages and disadvantages before acting; also to channel the human desire for revenge and prevent vigilante justice. 1) Problems: Different levels of pleasure, hard to determine the optimum level of sanctioning; assumes a rational actor; some studies show that raising the chance of being caught is more effective. b. Incapacitation Usually incarceration; forward looking; raises issue if punishing for future crimes. c. Retribution Punishment is the end itself; meant to carry blame and stigma, community condemnation; do not punish of not wrong or if no choice was made. Eye for an eye 1) Problems: a. Critique of agency or freedom; assumes people choose and all equally free in society; left wing critique (society must be equal before criminal justice system can be fully applied); b. Costs of punishing all crimes is too high for society; c. Provides no yardstick as deterrence does; says nothing of proper severity of punishment; will society tolerate what severe offenders deserve? d. Rehabilitation Defendant is to be punished so that he can be trained not to commit the crime again.

C.

Legality:
a) Principle of legality conduct must be specifically prohibited by criminal laws before it may be punished b) Serves many purposes: a. Provides notice of what conduct is illegal b. Confines the discretion os the police and their enforcement of the laws c. Prevents judges and juries from creating new crimes

D.

Conduct:
a) Most crimes consist of 4 elements: a. A prohibited act that results in some kind of social harm (actus reus, guilty act) b. Prohibited mental state (mens rea or guilty mind). c. A chain of causation that links the s actions with the social harm; and d. Concurrence between the mens rea and actus reus. 1) The criminal law generally requires an act in addition to a culpable mind-set before an individual can be held liable.

E.

Actus Reus
a) Act requirement that shows our philosophical commitment to autonomy, free speech, and liberty preservation; limits state interference; also shows solid proof of intent and actual harm; very few crimes involve only thoughts. b) The common law requires that all crimes require an act or omission in addition to a bad state of mind.

Criminal Law Levy II 2011


Mere thoughts are not enough. A conduct crime is based on prohibited conduct i.e. DUI (punishing a specific dangerous behavior, potential social harm.) a. A result crime is based on an outcome, e.g. murder (punishing an unwanted outcome, social harm). b. A criminal act must be voluntary, but a voluntary act leading to a loss of control may still result in criminal culpability (e.g. inebriation). MPC 2.01 (1)(2). 1) Purpose is so there is no punishment for thoughts alone d) Involuntary Acts a. Involuntariness is an actus reus defense not a mens rea defense. This defense can be made out whether or not accused had mens rea and whether or not crime is strict liability. b. MPC 2.01 (2) defines acts that it considers involuntary (largely drawn from common law): 1) reflex or convulsion; 2) bodily movement during unconsciousness or sleep; 3) conduct during hypnosis (or resulting from a hypnotic suggestion); and 4) (catch-all) bodily movement that otherwise is NOT a product of the effort or determination of actor, either conscious or habitual. e) Omission a. An omission of duty owed to another can sometimes be prosecutable, IF that duty is legally required. 1) In order to be held criminally liable for omission, the must be physically capable of acting. Additionally, the must have known the facts giving rise to the need for action. a. General rule: no duty to help even if duty excused if cannot help w/o harm to self b. About a legal duty NOT a moral obligation 2) Duties: a. Statute b. Contract individually made law; necessity of fulfilling enforceable contract is legal duty. c. Status Relationship Either applied by law or assumed; i.e. parents/child. d. Voluntary Assumption of Care secludes that helpless person from other people e. Creation of Peril where one person creates risk of harm to another. 3) Jones v. US babies covered in roaches, was there a duty? b. MPC 2.01(3) Liability for the commission of an offense may not be based on an omission unaccompanied by action unless: 1) The omission is expressly made sufficient by the law defining the offense; or 2) A duty to perform the omitted act is otherwise imposed by law. c)

F.

Status Crimes
a) Cannot be punished for status alone but can be convicted for an act related to the status. b) Acts are not states of being, have to be punished not for being a drunk but doing something while drunk. a. Robinson v. California crime = being an addict. Punish for disease = Cruel and unusual punishment b. Powel Crime = being drunk in public. Being in public is a voluntary act therefore an act

G.

Possession Crimes
a) Possession itself may be a crime, MPC 2.01(4) requires that the defendant knowingly procured or received the thing possessed or was aware that she is in control of the item illegally possessed and have sufficient time to terminate possession a. Have the ability to guide the drugs destiny b. Wheeler v. US heroin in apartment. Constructive possession and ability to guide its destiny c. People v. Ireland wife selling weed. He had no control over the weeds destiny b) Joint possession- contraband can be possessed by more than1 person simultaneously when it is found in a place where more than one person is shown to have been aware of its existence and to have control of it. c) Constructive possession- Can still be guilty even if it is not found on her immediate vicinity. Possessed constructively when the accused is shown to have been aware of the existence and to have exercised control over it.

H.

Mens Rea 2

Criminal Law Levy II 2011


a) Establishes what D must have thought at the time the act was committed for criminal liability to attach. The intent required for each crime ordinarily is part of the definition of that crime and may vary with regard to the different elements of the crime. a. Motive refers to Ds reason for acting and generally is not essential for criminal liability. b. Narrow or elemental definition is a crime committed with a specific level of intent as specified in the statute. b) Common Law Approach: a. Maliciously: Def. realizes the risk his conduct creates and engages anyway (recklessly under MPC) b. Intentionally: Being aware of the harm she is likely to cause, although harm is not primary aim OR actually harming with the intent to harm c. Negligently: not using a reasonable standard of care under the circumstances d. Willfuly: acting with purpose to violate the law c) Approach to Mens Rea Under MPC 2.02 a. Minimum Requirements of Culpability - Except as provided in Section 2.05, a person is guilty of an offense unless he acted purposely, knowingly, recklessly, or negligently, as the law may require, with respect to each material element of the offense. b. Purposely: A person acts purposely with respect to his conduct when it is his conscious object to engage in a certain conduct or cause a certain result. Intent c. Knowingly: A person acts knowingly with respect to the nature of his conduct when he is aware that his conduct will necessarily or it is practically certain that such a result will occur. a. US v. Villegas blood in vials did not know would cause hepatitis 2) Conduct performed knowingly usually also satisfies the mental state of a statute that requires willful blindness or Ostrich defense. 3) Some jurisdictions hold that willful blindness, which can be defined as a purposeful avoidance of knowledge in a reckless manner, is the same as knowledge. 4) A person who has suspicions aroused, but purposely disregards such suspicions and does not make further inquiries, and is aware to a high-probability of the truth, can be considered as equally culpable as one with knowledge. 5) Under the MPC, you must know that it is highly probable that your suspicion is true and must not believe that it is false in the instant case. MPC 2.02 (7)-(8). d. Recklessly: A person acts recklessly with respect to their conduct when he consciously disregards a substantial or unjustifiable risk that circumstances exist or that a prohibited result will follow, and this disregard constitutes a gross deviation from the standard of care that a reasonable person would exercise in the situation. 1) An act performed recklessly is also usually performed wantonly. 2) Mere realization of the risk is NOT enough. He must know that the injury might result. Thus, reckless involves both objective (unjustifiable risk) and subjective (awareness). e. Negligence: A person acts negligently when he fails to be aware of the substantial and unjustifiable risk that circumstances exist or a result will follow, and such failure constitutes a substantial deviation from the standard of care that a reasonable person would exercise under the circumstances. 1) To determine whether a person acted negligently, a purely objective standard is used. However, it is not the reasonable person standard that is used in torts; the defendant must have taken a very unreasonable risk in light of the usefulness of his conduct, his knowledge of the facts, and the nature and extent of the harm that may be caused. d) Motive Distinguished: a. A motive for a crime is distinct from the intent to commit it. b. A motive is the reason or explanation underlying the offense. c. It is generally held that motive is immaterial to substantive criminal law. A good motive will not excuse a criminal act. 1) On the other hand, a lawful act done with bad motive will not be punished. 2) Example: An impoverished woman steals so that her hungry children may eat. Despite her noble motive, feeding her children, the woman could be held criminally liable for her acts because her intent was to steal.

Criminal Law Levy II 2011 I. MPC Rules of Construction


a) A person is not guilty of a crime, with the exception of a narrow class of strict liability offense, unless he acted purposely, knowingly, recklessly, or negligently as the law may require. b) State of mind applies to all material elements of the offense. Often a statute will establish a culpable state of mind without indicating it is required for all the material elements of the offense. In that case specific state of mind applies to all material elements of the offense unless a contrary purpose appears in the statute. a. Example: Under a statute imposing criminal liability on anyone who knowingly makes a sale of an intoxicating beverage to a minor, the MPC would require knowledge for each material element of the offense. Thus, if the defendant can show that she did not know a sale took place, that the beverage was intoxicating, or that the purchaser was a minor, she will be able to avoid liability. c) If the statute defining the offense (other than strict liability) does not include a state of mind requirement, the defendant must have acted with at least recklessness with regard to each material element of the offense. d) Because a standard of recklessness is assumed where the state of mind is not specified, if a lower state of mind will satisfy liability, or if a higher state of mind is required, those standards must be indicated in the language of the statute. a. Example: Under a statute creating criminal liability for anyone who sells intoxicating beverages to whom he should know to be a minor the material elements including the act of selling and the attendant circumstances that the beverage be intoxicating and that the purchaser be a minor. Under the MPC formula, a minimum standard for recklessness is required as the state of mind for the first two elements, while the third element of the statute specifies that only a negligence level of fault is required.

J.

Statutory Interpretation:
a) b) c) d) Conduct element: the action that must be performed. Result element: the result that must occur. Attendant circumstances: what other features must be present. Three Components: a. Text: The most important; if the text is clear, go with the text. b. Legislative History: Legislative intent at the time the statute was enacted. c. Present Day (issues, specifications, etc.): Layer the previous two components with present day circumstances; this is the least embraced component. Judges goal in interpreting a statute are: a. Respecting the plain language of the statutory text. b. Decide what the intent of the legislature or voters was when they formed or passed a statute. c. Be consistent with past rulings. Judges have informal rules for interpreting statutes: a. How should a judge prioritize when reading a statute? b. Is this statute or its interpretation enforcement efficient or constitutional? c. What rules of grammar and syntax should be used in interpreting statutes?

e)

f)

K.

Statutory Construction:
a) Look to the plain language of the statute b) Ways to look at statutes a. Intentionalist- Chooses the outcome that the legislature would have chosen had it seen the case. Attempt to reconstruct how legislatures thought at the time. b. Textualist- Just look at the words, what they wrote is the law. Does not always work because of ambiguity in the words. c. Pragmatist- Look at the text but still use common sense, respect for precedent, and a sense for societys needs c) Canons a. Substantive 1) Thumb on the scale- Chose the interpretation most consistent with common law. 2) Rule of lenity- If the criminal statute does not clearly outlaw private conduct, then the private actor cannot be punished.

Criminal Law Levy II 2011


3) Presumption that US statutes have no extraterritorial effect, unless the intended effect is clearly mentioned in the statute. 4) Strong canon against retroactive effect 5) Statutes avoid creating serious constitutional questions b. Linguistic 1) Punctuation rules akin to grammar 2) Expressio unius est exclusion alterius - The expression of one thing excludes all others 3) Ejusdem ganeris - Of the same kind 4) Noscitur a sociis - Known by its associates 5) Rule against surplusage c. If one meaning of a statute is duplicated by another statute then it is an incorrect interpretation d) Due Process requires that a criminal statute give fair warning of the conduct that it makes a crime. Before a man can be punished as a criminal under the Federal Law, his case must be plainly and unmistakably within the provisions of some statute. Due Process requires that the prosecutor to persuade the fact finder beyond a reasonable doubt of every fact necessary to constitute the crime charges a. Keller v. Superior Court man killed fetus No fair warning because a fetus was not considered a human being under the statute b. Opposite of Shaw v. director of Public Prosecutions Courts believed they had a duty to protect public morals and made law dissent said no fair warning

L.

Transferred Intent:
1. 2. The s guilt is thus exactly what it would have been had the blow fallen upon the intended victim instead of the bystander. If D intends to harm one person or item of property and does similar harm to another person or item, D will be treated as if she had, in fact, intended the result that occurred (the intent is transferred) and criminal liability will attach. MPC: if D causes less harm than intended, he will only be responsible for the harm resulted. CL: Responsible for higher crime. Specific Intent: If the definition of a crime requires NOT ONLY the doing of an act, BUT the doing of it with a specific intent, the crime is a specific intent crime. This definition is focused on the consequence. It is necessary to identify specific intent crimes for 2 reasons: a. Need for proof: the existence of specific intent crime cannot be inferred from the doing of the act. The prosecution must produce evidence tending to prove the existence of the specific intent. b. Applicability of defense: some defenses, such as voluntary intoxication, drugged condition and unreasonable mistake of fact, apply only to specific intent crimes. c. Major Specific Intent Crimes: i. Solicitation: intent to have the person solicited commits the crime. ii. Attempt: intent to complete the crime iii. Conspiracy: intent to have the crime completed. iv. First degree premeditated murder: (where defined by statute) premeditated intent to kill. v. Assault: intent to commit a battery. vi. Larceny and Robbery: intent to permanently deprive another of his interest in the property taken. vii. Burglary: intent at the time of entry to commit a felony in the dwelling of another. viii. Forgery: intent to defraud. ix. False Pretenses: intent to defraud x. Embezzlement: intent to defraud. General Intent: a. Intent to commit the act that causes the harm, not the intention of the consequences of her acts. Generally, all crimes require general intent, which is an awareness of all factors constituting the crime. i. The must be aware that she is acting in the proscribed way and that any attendant circumstances required by the crime are present. (Note that the NEED NOT be certain

3.

M.

Distinguishing between Specific & General Intent, Malice Crimes, & Strict Liability:
1.

2.

Criminal Law Levy II 2011


that those attendant circumstances exist; it is sufficient that she is aware of a high likelihood that they exist. b. Major general intent crimes: i. Battery ii. Rape iii. Kidnapping iv. False imprisonment c. When the definition of a crime consists of only the description of a particular act, without reference to intent to do a further act or achieve a future consequence, we ask whether the intended to do the proscribed act. Definition is focused only on description of the act. d. Silence on the mens rea in the definition of the offense DOES NOT mean that no mens rea is required. i. A jury can infer the required intent merely from the doing of the act. It is NOT necessary that evidence specifically providing the general intent be offered by the prosecution. e. Various criminal statutes might also use words such as willfully, deliberately, or feloniously to describe the requisite state of mind. i. Again, these terms have no generally agreed upon meaning, but are often equated with general intent. ii. Sometimes, however, they will be construed as requiring an unusually significant mental state. f. There is no distinction between general and specific intent in the MPC. Malice Crimes: a. Some crimes require malice, which is usually construed as a requiring the commission of a volitional act without legal excuse or justification. Ill will or hatred need not be shown. b. Major malice crimes: i. Common law murder ii. Arson c. The common law created this special mental state category to deny murder and arson the specific intent defenses. i. To establish malice in these cases, the prosecution need ONLY show that the recklessly disregarded an obvious high risk that the particular harmful result would occur. Strict Liability: a. A strict liability offense is one that does not require awareness of all of the factors constituting the crime. 1. Generally, the requirement of a state of mind is NOT abandoned with respect to all the elements of the offense, but only with regard to one or some of the elements. b. Major strict liability crimes: 1. Statutory rape 2. Selling liquor to minors 3. Bigamy (some jurisdictions) c. The major significance of a strict liability offense is that certain defenses, such as mistake of fact, are not available. d. Two types of strict liability offenses: public welfare offenses are generally regulatory offenses, i.e. offenses that are part of the regulatory scheme or morality offenses. 1. They generally involve a relatively low penalty and are not regarded by the community as involving significant moral impropriety. 2. If the punishment of the wrongdoer far outweighs regulation of the social order, mens rea is required. However, if the penalty is light involving no imprisonment, mens rea is probably not needed. e. Note that the mere fact that a statute is silent on the question of mental state does not necessarily mean that the offense is a strict liability offense. 1. If no mental state is expressly required by the statute, the courts may still interpret the statute as requiring some mens rea, especially if the statute appears to be a codification of a

3.

4.

Criminal Law Levy II 2011


traditional common law offense or if the statute imposes a severe penalty. Strict liability may expressly state that there is no mens rea 2. Example: Federal regulation prohibits the transfer of firearms not registered under federal law. Is it a defense that the was ignorant of the fact that a firearm was not registered? 1. No, because this is a strict liability offense. Awareness of the fact of nonregistration is not necessary, although it is necessary that the be aware of the fact that she possessed a firearm. f. Public Welfare Offenses: Traffic violations, improper handling of food 1. US v. Freed possession of unlicensed grenade finding out possession was not an innocent act would not surprise one. 2. No direct or immediate injury to person or property 3. Create a danger which law seeks to minimize 4. Penalties are small 5. Injury is the same in these offenses 6. Statutory Violations that are purely Regulatory in Nature and involve widespread injury to public. 7. 6.) Primary purpose is regulation rather than punishment g. Factors that Courts use to Interpret Criminal Statutes to Determine Strict Liability: 1. Legislative History and Context 2. Guidance from other statutes 3. The severity of the punishment 4. The seriousness of the public harm the statute seeks to prevent 5. The Ds opportunity to ascertain the true facts 6. The difficulty of proving mental state 7. The number of prosecutions to be expected under the statute h. MPC 1.04(5) has rejected strict liability except in cases of violations that cannot result in prison time or criminal status. Vicarious Liability: a. Vicarious liability exists when one person is held guilty of a crime committed by another simply because of the relationship between two persons. No affirmative participation is required. b. MPC 2.06(1)-a person is guilty of an offense if it is committed by his own conduct or by the conduct of another for which he is LEGALLY accountable or both. c. MPC 2.06(2)(b)- legally accountable when he is made accountable for the conduct of such other person by the code or by the law defining the offense. d. The responsibility of defendant for the criminal acts of another person, ordinarily without a showing that the defendant had a culpable mens rea. Vicarious liability is usually limited to employeremployee relationships. In all cases, legislative intent determines whether to apply vicarious liability.

5.

N.

Mistake:
a) Mistake is a way of trying to negate the mens rea element of a criminal offense. i. The common law approach to mistake and ignorance relies on the distinction between specific and general intent. ii. A second distinction is whether the mistake is a mistake of fact or a mistake of law. b) Mistake or Ignorance of Fact: a. Ignorance or mistake as to the matter of fact will affect criminal guilt only if it shows that the did not have the state of mind required for the crime. b. Common Law 1) Specific intent crimes - a mistake of fact is a defense to a specific intent crime when the accused could establish simply that he or she honestly believed in a mistaken circumstance that negates the mens rea required for the offense charged (a subjective approach). 2) General intent crimes - mistake of fact can be a defense to a general crime only when the accused could establish BOTH that he honestly believed in a mistaken circumstance AND that such mistaken belief was reasonable as well (an objective approach). 3) Strict liability crimes - ignorance or mistake of fact is ordinarily no defense to a strict liability crime. However, a number of courts have implied an intent requirement where D offers

Criminal Law Levy II 2011


evidence of a reasonable mistake. Furthermore, some courts unwilling to find a mens rea requirement have still sustained a defense of reasonable mistake. c. Minority Jurisdiction - A few decisions have suggested that the mistake must have been such that if the situation had been as D believed it to be, her actions would have been both legally and morally permissible. d. MPC 2.04(1) defense if the ignorance or mistake of fact negates the purpose, knowledge, belief, recklessness or negligence required to establish a material element of the offense or the law provides that the state of mind established by such ignorance or mistake constitutes a defense. e. MPC 1.13(10) the facts need to be a MATERIAL element. f. If the mistake or ignorance is offered to negate the existence of malice, it MUST be a reasonable mistake or ignorance, i.e. the type of mistake or ignorance that a reasonable person would have made under the circumstances. 1) Note that some jurisdictions will impose a liability even if a reasonable mistake is made. 2) Has to be a mistake critical to the crime g. Mistaken but still guilty: can a defendant assert a mistake of fact defense if she genuinely made a mistake as to the charged crime but would still be guilty of another crime 1) MPC guilty of lessor offense committed 2) Common Law guilty of the higher offense. h. Gordon v. State: defendant honestly believed he was old enough to vote, however, iff he voted recklessly or carelessly, not in good faith, when the facts are doubtful or uncertain, ignorance is no excuse. i. Analysis: 1) Is it general intent, specific intent, or a strict liability crime? a. Strict liability: intent does not matter, therefore, mistake doesnt matter. b. Specific intent crime: decide if the mistake relates to the specific intent portion of the crime, and if so, does the mistake prevent the actor from having the requisite specific intent? c. General intent crime: apply a culpability analysis Was the defendants state on mind blameworthy, and whether their mistake, and in turn their behavior, was reasonable. Mistake or Ignorance of Law a. The general rule is that it is not a defense to a crime that the was unaware that her acts were prohibited by the criminal law or that she mistakenly believed that her acts were not prohibited. This is true even if her ignorance or mistake was reasonable. b. Common law - mistake of law is almost always not an excuse in general intent crimes, but may be an excuse in specific intent crimes. 1) In general intent, the error must be both honest and reasonable. 2) With specific intent, the error must be simply honest. a. People v. Wendt did not file taxes knew if he did not file them they would not be filed. b. US v. Baker: mistake of law only avails if it is objectively reasonable. Tried to say it was like responding to an officers call which is an exception under the MPC but it does not apply to voluntary acts for the military. c. MPC 2.04 - A mistake of law is a defense if the mistake negates the mens rea of the offense, or the law provides that the state of mind established by such a mistake constitutes a defense. The defense must be made based on reasonable reliance under subsection (3) or due to lack of notice. d. Exceptions: 1) When the mistake of law is just like a mistake of fact: defendant lacks the mens rea required because the statute requires the defendant to know what the law requires. 2) When the defendant is mislead by official authority a. Once the official statement of law is corrected one may not have this defense b. Mislead either by reliance on judicial decision, invalid statute, administrative order, or in rare situations reliance of an official interpretation of the law

c)

Criminal Law Levy II 2011


3) Due process - When there has been no reasonable notice of the law: lambert defense, register as felon in CA. a. MPC if law abiding and prudent person would not have learned of the laws existence, defendant may argue ignorance of the law. 4) Mistake is based on reliance of a statute that is found to be invalid. e. Unlike reasonable reliance on an official interpretation of the law, relying on the advice of ones own counsel is normally not allowed as a true affirmative defense to a crime. 1) If, however, the reliance on the attorney negates an otherwise necessary material element (e.g. knowingly violating the law), such reliance can demonstrate that the government has not proved its case beyond a reasonable doubt. f. Disagreement with the law is no defense g. Cultural Defenses no formal cultural defenses if person chooses to live in US obey US law

O.

Intoxication
a) You CAN form intent while intoxicated, EVEN IF it is specific intent. You may not be able to make good decisions while intoxicated, but you can still form intent. b) Evidence of intoxication, under the MPC, may be raised whenever the intoxication negates an element of the crime. c) Defense to specific intent crimes: Voluntary intoxication evidence may be offered when the defendant is charged with a crime that requires purpose or knowledge to establish that the intoxication prevented the defendant from forming the requisite intent. a. Thus, it may be a good defense to specific intent crimes, but will usually not be a sufficient defense to general intent crimes. The defense is not available if the defendant purposely becomes intoxicated in order to establish the defense. b. Voluntary intoxication is NOT a defense to crimes requiring malice, recklessness, negligence, or crimes of strict liability. 1) MPC 2.08(1) refuses to recognize voluntary intoxication as a defense unless it negatives an element of the offense. a. Intoxication is not a defense unless it negatives an element of the offense b. Intoxication is not a defense to recklessness c. Intoxication, does not in itself, constitute mental disease c. Involuntary Intoxication: is universally recognized as a defense if it negates the mens rea required for the crime 1) Unwitting intoxication: unaware of spiked drink 2) Coerced intoxication: forced to ingest 3) Pathological effect intoxication: medication or alcohol producing grossly excessive effect d. Degree of intoxication: must be to the extent that a person could be considered mentally incapable

P.

Theft
a) Consolidation: the MPC consolidated into one theft crime most common separate crimes at common law into one, inclusive theft offense. a. MPC 223.2 it is theft by unlawful taking or disposition if a person unlawfully takes, or exercises unlawful control over, movable property of another with purpose to deprive him thereof or unlawfully transfers immovable property of another or any interest therein with purpose to benefit himself or another not entitled thereto. b) Larceny a) Common law - larceny consists of an intentional taking and carrying away of tangible personal property of another by trespass with intent to permanently (or for an unreasonable time) deprive the person of his interest in the property. b) MPC 223.1 - sufficient if the defendant exercises unlawful control over moveable property. c) Taking and carrying away 1. use does not equal taking and carrying away 2. Asportation (carrying away): bare removal from the place in which he found the goods even if the thief does not quite make off w/ them. Slight movement is still larceny. 3. Caption (taking): severance of the goods from the possession of the owner d) Trespassory taking- no consent

Criminal Law Levy II 2011


When the person who lawfully possesses the property does not consent to the taking MPC = exercises unlawful control over the property a) Putting a for sale sign on something e) Property in the possession of another 1. Common law - Tangible personal property 2. MPC 223.0(6) anything of value from real estate to tangible property 3. Custody does not equal possession a) Generally, a bailee has possession. If however, she opens closed containers in which the property had been placed by the bailor, the possession is regarded by use of a fiction as returning to the bailor. If a bailee misappropriates property after breaking bulk, she takes it from possession of the bailor and is guilty of larceny if she has the intent to steal. 4. Abandoned property belongs to nobody 5. Lund v. commonwealth scrap paper does not equal property 6. MPC 223.0(7) - property of another includes property in which any persons other than the actor has an interest which the actor is not privileged to infringe regardless of the fact that the actor also has an interest in the property a) Henry v. State property to secure debt is property of another. 7. MPC if a person finds property and knows it belongs to another, he must make reasonable efforts to return the item. f) Intent to permanently deprive owner 1. Borrowing is insufficient for common law crime, must permanently deprive owner. 2. If take it with the intent to keep it then later changes his mind and decides to return it, still guilty of larceny 3. Oxford v. Moss: stole exam paper no intent to permanently deprive 4. Obtaining services cannot give rise to larceny. Except in MPC 223.7 5. Intangibles cannot give rise to larceny. a) Gas and electric are considered tangible goods. g) Larceny is a crime against possession therefore; all that is necessary is that the property be taken from someone who has possessory interest superior to that of the . h) It is essential that the actually obtain control of the property. 1. Defendant cannot be guilty of larceny if it is an abandoned property. 2. Mere destruction of the property is not sufficient to constitute a taking. Even if a obtains control of the property through the act of an innocent agent, it is a taking. 3. It is not larceny to take money or goods of another if the honestly believes that she is entitled to repayment for a debt of another (although the goods must not be worth more than the amount of the debt). a) In this situation, the believes the property is hers and therefore lacks intent. 4. If the property taken is not for sale, the fact that the intends to pay the other for it does not negate the larceny. a) If the property is for sale and the has a specific and realistic intent to repay the person, the taking is not larceny. Embezzlement a) When a person fraudulently converted the property of another person while he or she was in lawful possession of the property. Happens when the bailee who starts to use the property he is supposed to be safeguarding. 1. Specific intent crime 2. No permanently - even if one eventually gives the property back still embezzlement b/c he converted it fraudulently and wrongfully took possession of another. b) Common Law - the fraudulent appropriation of property of another by one who has been entrusted with possession c) Elements 1. A fraudulent Use is not consistent with the use entrusted to you 1. 2.

c)

10

Criminal Law Levy II 2011


Conversion/appropriation of Appropriation = severing of ownership 3. The property of another 4. That the defendant already possesses d) Claim of right stands up to property of another e) Money paid to another is no longer property of the first that is subject to embezzlement f) This was not originally a common law crime. Intended to plug the gaps in the law of larceny, it was created by statute and is regarded as a common law crime now. 1. Modern statutes often distinguish between grand and petit embezzlement (a felony and a misdemeanor) based upon the value of the property embezzled. 2. Embezzlement usually requires the fraudulent conversion of property of another by a person in lawful possession of that property. g) In embezzlement, the misappropriation of the property occurs while the has lawful possession of the property. In larceny, it generally occurs at the time the obtains wrongful possession of the property. 1. Embezzlement requires intentional conversion. a. Conversion requires only that the deal with the property in a manner inconsistent with the trust arrangement pursuant to which he holds it. i. The conversion need not result in direct personal gain to the . 2. Embezzlement requires that the property converted be that of someone other than the converter. Therefore, a person who borrows money, converts it to his own use, and subsequently fails to repay it is not guilty of embezzlement. h) A must intend to defraud for a conversion to become embezzlement. This appears to be the functional equivalent of larcenys specific intent to permanently deprive. 1. If the intended to restore similar or substantially identical property, it is embezzlement, even if its money of identical value he intended to restore. 2. As in larceny, embezzlement is not committed if the conversion is pursuant to a claim of right to the property. i) If it is clear that there has been has been a conversion of property, the victim need not make a demand that it be returned. d) Lost or mislaid property a) When a person finds goods that have actually been lost, and takes possession w/intent to appropriate to own use really believing, at the time, or having good ground to believe, that the owner can be found it is larceny. b) How much diligence must be used 1. Court does not specifically care about amount of diligence used just that he did know that he did know that there was an owner and he did not try to find them 2. Brooks v State found money and did not look for the owner. c) Common Law: 6 elements of larceny plus: a. Had a clue suggesting the owner could reasonably be found b. Intent to permanently deprive owner at the time property was taken. d) MPC 223.5 - Theft of Property Lost, Mislaid, or Delivered by Mistake: A person who comes into control of property of another that he knows to have been lost, mislaid, or delivered under a mistake as to the nature or amount of the property or the identity of the recipient is guilty of theft if, with purpose to deprive the owner thereof, he fails to take reasonable measures to restore the property to a person entitled to have it. e) Mistaken Delivery a) US v. Rogers bank teller gave Rogers too much money. 1. Essential elements of the offense are a) that the defendant knew when he received the money from the teller or picked it up from the counter that it was more than his due and b) that he took it from the bank with the intention of converting it. b) If the transferee knowing of the transferors mistake receives the goods with the intention of appropriating them 2.

11

Criminal Law Levy II 2011


1. 2. However, if innocent at the time of initial receipt of overpayment its subsequent conversion by him cannot be larceny MPC a crime can be committed even if the defendant formed the intent to deprive at some point after the time of mistaken delivery Unilateral mistake = larceny (intent at time of taking) Mutual mistake = no larceny

f)

g)

e)

3. 4. False Pretenses: 1. Elements: a) A misrepresentation by the defendant (puffing about value of an item is not misrepresentation) b) Of a past or present material fact c) With the intent to defraud the victim d) Where the victim relies on the misrepresentation e) In transferring TITLE to some property 1. Chaplin v. US liquor stamps Larceny by Trick 1. Elements: a) Taking POSSESSION (not title) of the property of another b) By knowingly making false representations as to material facts or making false promises c) With an intent to defraud i. MPC - No intent to defraud if the defendant pays or intended to pay for the goods (if the good was for sale in the first place) 2. Mason v. State stealing the keg but wanted to pay Defenses to theft crimes: a. Defendants most often claim 1) The alleged property does not qualify as property under the law 2) The owner consented to the taking 3) The defendant did not intend to permanently deprive the owner but was simply borrowing the object without permission. a. Not a defense if formed AFTER the act 4) May also allege a claim of right

Q.

Aggravated Property Crimes and other


a) Robbery a. A felony in all jurisdictions, robbery consists of the following: 1) a taking of personal property of another 2) from the others person or presence 3) by force or intimidation 4) with the intent to permanently deprive him of it. b. Thus robbery is basically an aggravated form of larceny in which the taking is accomplished by force or threats. c. Common law: Robbery was larceny that included a taking from the person or in the presence of the victim that was accomplished by violence or threat of violence. d. State v. Mejia shot Garcia trying to get his money back e. MPC in course of committing theft he: a. Inflicts serious bodily injury or uses force upon another; or b. Threatens another w/ or purposely puts him in fear of his immediate serious bodily injury; or st nd c. Commits or threatens immediately to commit any felony of the 1 or 2 degree f. MPC 223.1(3) - recognizes an affirmative defense to theft if the D reasonably believed that he was recovering property that was owed to him. g. Armed Robbery = robbery while in possession of a dangerous weapon b) Burglary a. At common law: 1. Breaking 2. Entering 3. The dwelling house

12

Criminal Law Levy II 2011


4. Of another person 5. At night time 6. With intent to commit felony b. Modern: 1) unauthorized entering 2) of a building or structure 3) w/ the intent to commit any crime c. People v Gauze does not count if entering own house d. Permission to enter a store is conditional upon a purpose consistent with the reason the business is open. c) Extortion: Extortion is an offense that generally has been expanded by modern statutes far beyond its initial common law definition. a. Common Law - Collection of Unlawful Fee: the common law misdemeanor of extortion consisted of the corrupt collection of an unlawful fee by an officer under color of his office. b. Modern Definition - Blackmail, defined as obtaining property from another by means of certain oral or written threats. a. The prohibited threats often include threats to do physical harm to the victim or others, or threats to damage the victims property. b. Under some statutes, the crime is completed when the threats are made with the intent to obtain money or something of value; the threat is the essence of the intent. d) Receipt of Stolen Property: The common law misdemeanor of receipt of stolen property in substantially identical to the modern offense. a. The elements of the crime are receiving possession and control of stolen property known to have been obtained in a manner constituting a criminal offense by another person with the intent to permanently deprive the owner of his interest in the property. b. Manual possession of the property, while sufficient for receiving, is not necessary. a. It is also receiving if the thief places the stolen property in a place that the has designated or for profit, the arranges for a sale of the property by the thief to a third party. b. Most jurisdictions define stolen property broadly to include property obtained by commission of any of the property offenses. 1) However, the property must have stolen status at the time it is received by the . e) Forgery: At common law, forgery and uttering a forged instrument were separate offenses. a. Forgery consists of making or altering a false writing with intent to defraud. Uttering a forged document consists of offering as genuine an instrument that may be the subject of forgery and is false with intent of defraud. f) Malicious Mischief: The common law misdemeanor of malicious mischief consists of malicious destruction of, or damage to, property of another. a. Destruction of the property is not required for malicious mischief. 1) All that is necessary is that some physical damage be done that impairs the utility of the property or materially diminishes its value. g) Assault: an attempted battery a. Actus Reus any act beyond words b. Mens Rea specific intent crime because it requires that the defendant intended to harm victim or reasonable apprehension h) Battery: the unlawful application of force to another person resulting in either bodily injury or an offensive touching general intent because only need the intent to touch i) Arson: Burning (the act) the dwelling house (now includes commercial buildings) of another with malice (mens rea) pg 49 of G.G. j) Kidnapping: intentional unlawful confinement, movement of the victim, threat or use of force or deception must be w/o victims consent Homicide: a) Common Law:

R.

13

Criminal Law Levy II 2011


Categories of Homicide Murder Mens rea requirement
Malice aforethought: 1. Intent to kill; 2. Intent to inflict great bodily injury; 3. Reckless indifference to a high risk to human life (abandoned and malignant heart); or 4. Intent to commit a felony (felony murder). 1. It was a provocation that would arouse sudden and intense passion in the mind of an ordinary person 2. the defendant was in fact provoked; 3. there was not sufficient time between the provocation and the killing for passions of a reasonable person to cool; and 4. the defendant in fact did not cool off between the provocation and the killing.

Voluntary Manslaughter

Involuntary Manslaughter

1. committed with criminal negligence (defendant was grossly negligent) or 2. during the commission of an unlawful act (misdemeanor or felony not included within the felony murder rule).

a.

b.

c.

Murder is defined as the unlawful killing of another human being with malice aforethought. Malice may be express or implied and is not ill-will or hatred. 1) This includes four states of mind: a. Intent to Kill: which includes the equivalent of MPC purpose and knowledge. b. Intent to Cause Grievous Bodily Harm: knowledge or purpose is sufficient for murder if victim actually dies. c. Extreme Recklessness: depraved heart murder unintentional homicide under circumstances evincing a depraved mind or an abandoned and malignant heart; equivalent to recklessness with extreme disregard for human life. d. Intent to Commit a Felony: felony murder doctrine. 2) Knowing or Purposeful homicide was murder unless committed in the heat of passion engendered by adequate provocation Manslaughter. 3) Defense to Murder: In many jurisdictions, in addition to the provocation defense, voluntary manslaughter can also be established by imperfect self-defense where an accused person honestly believed that he or she needed to kill in order to take justified protective action but such belief of the accused was unreasonable. 4) Deadly Weapon Rule: Intentional use of a deadly weapon authorizes a permissive inference of intent to kill. A deadly weapon is any instrument, or in some limited circumstances, any body part, used in a manner calculated or likely to produce death or serious bodily injury. Manslaughter: Unlawful killing of a human being without malice; divided into voluntary (upon a sudden quarrel or heat of passion) and involuntary (in the commission of an unlawful act not amounting to a felony, or in the commission in an unlawful manner, or without due caution and circumspection, o a lawful act which might produce death.) Voluntary manslaughter is the intentional homicide in the heat of passion, with adequate provocation, and occurring before an adequate time to cool off therefore lacking the malice necessary to establish murder. 5) Note that there is NO malice aforethought required for manslaughter. 6) Provocation: There must be: a. The provocation must have been one that would arouse sudden and intense passion in the mind of a reasonable person such as to cause him to lose his selfcontrol; b. The defendant must have, in fact, been provoked. c. without a cooling off period for a reasonable person, and d. The defendant did not cool off. 7) Most frequently recognized are in cases of being subjected to serious battery or a threat of deadly force; and discovering ones spouse in bed with another person.

14

Criminal Law Levy II 2011


8) Provocation inadequate as a matter of law: At common law, some provocations were defined as inadequate as a matter of law. The most significant was mere words. Modern courts tend to be more reluctant to take such cases from juries and are more likely to submit to the jury the question of whether mere words or similar matters constitute adequate provocation. 9) MPC: Actual provocation- passion is typically rage but can also be fear or any violent and intense emotion sufficient to dethrone reason d. Involuntary Manslaughter is an unintentional killing that is committed without malice. There are two types; 1) Criminal Negligence: If death is cause by criminal negligence, the killing is involuntary manslaughter. Criminal negligence requires a greater deviation from the reasonable person standard than is required for civil liability. a. Some states require that the have had a subjective awareness of the risk. This would then require that the was reckless in order to be guilty of involuntary manslaughter. b. Some states will criminalize this mental state, criminal negligence, as negligent homicide or criminally negligent homicide. It can be a crime other than involuntary manslaughter. 2) Reckless Manslaughter: Recklessly causing the death of another. a. Must be: 1. Aware of risk, and 2. Disregard that risk. b. Subjective and Objective: i. Substantial: Likely to get harm How bad the harm will be ii. Unjustifiable: Nature and purpose of conduct Risk e. Unlawful Act Manslaughter: A killing caused by an unlawful act is an involuntary manslaughter. 1) Misdemeanor Manslaughter: A killing in the course of the commission of a misdemeanor is manslaughter although most courts would require either that the misdemeanor be: i. Malum in se -- an inherently wrongful act (crimes that are dangerous or immoral, or ii. Malum prohibitum -- that the death be a foreseeable and natural consequence of the unlawful conduct. Malum prohibitum in general = Violate a specific prohibition of the law, not necessarily one that carries a moral violation 2) This rule was rejected by MPC and has been abolished and many states. f. Unintentional Killings. Three levels: 1) Uncontestable fault and civil liability but no criminal. 2) Gross deviation from the standard of care representing a wanton and reckless disregard for human life [involuntary manslaughter]. 3) An act so reckless that they manifest a wanton indifference to human life, depraved heart murder. b) MPC 210.00: Levels of homicide Murder Manslaughter Negligent Homicide Causing or Aiding Suicide Mens rea requirement
Purposely, knowingly, or acting with grossly reckless regard for human life Recklessly or under extreme emotional disturbance Negligently Purposely with force, duress or deception.

15

Criminal Law Levy II 2011


a. b. MPC does not distinguish between degrees of murder. 210.00 Definitions 1) In Articles 210-213, unless a different meaning plainly is required: a) human being means a person who has been born and is alive; b) bodily injury means physical pain, illness or any impairment of physical condition; c) serious bodily injury means bodily injury which creates a substantial risk of death or which causes serious, permanent disfigurement, or protracted loss or impairment of the function of any bodily member or organ; d) deadly weapon means any firearm or other weapon, device, instrument, material or substance, whether animate or inanimate, which in the manner it is used or is intended to be used is known to be capable of producing death or serious bodily injury MPC 210.1 Criminal Homicide a) A person is guilty of criminal homicide if he purposely, knowingly, recklessly, or negligently causes the death of another human being. b) Criminal homicide is murder, manslaughter, or negligent homicide MPC 210.2 Murder a) Except as provided in 210.3(1)(b), criminal homicide constitutes murder when a) It is committed purposely or knowingly; or b) It is committed recklessly under circumstances manifesting extreme indifference to the value of human life. Such recklessness and indifference are presumed if the actor is engaged or is an accomplice in the commission of, or an attempt to commit, or flight after committing or attempting to commit robbery, rape or deviate sexual intercourse by force or threat of force, arson, burglary, kidnapping or felonious escape. b) MPC definition of murder does not include malice. MPC uses their own language of culpability to described when a killing is serious enough to constitute murder. i. Reckless = subject aware of the risk ii. Negligent = should have known the risk but did not MPC 210.3 Manslaughter a) Criminal homicide constitutes manslaughter when: a) It is committed recklessly; or b) A homicide which would otherwise be murder is committed under the influence of extreme emotional or mental disturbance for which there is reasonable explanation or excuse. The reasonableness of such explanation or excuse shall be determined from the viewpoint of a person in the actors situation under the circumstances as he believes them to be. b) Manslaughter is a felony of the second degree. c) MPC 210.3(1)(b) a. Extreme emotional disturbance replaced heat of passion. Defines manslaughter to include homicide which would otherwise be murder committed under influence of extreme mental or emotional disturbance for which there is reasonable explanation or excuse. b. May involve a series of events rather than one precipitating event. c. Two components: 1. Must have acted under the influence of extreme emotional disturbance, and 2. Must have been a reasonable explanation or excuse, the reasonableness of which is to be determined from the viewpoint of a person in the s situation under the circumstances as the believed them to be.

c.

d.

e.

16

Criminal Law Levy II 2011


Interpreted to mean that a jury or judge should make the decision based on the internal situation of the and the external situation as he perceived it, but applying the standards of reasonableness to such provocation. d) MPC 210.3 & 210.4 define manslaughter and negligent homicide dependent on mens rea of person causing the unintentional killing. f. MPC 210.4 Negligent Homicide a) Criminalize the separate offense of negligent homicide. Where involuntary manslaughter requires proof of recklessness, the separate crime of negligent homicide creates a lesser level of homicide which applies to killings that occur merely as a result of an accused persons criminal negligence. a. Criminal homicide constitutes negligent homicide when it is committed negligently. b. Negligent homicide is a felony of the third degree. g. MPC 210.5 Causing or Aiding Suicide: A person may be convicted homicide for causing another to commit suicide only if he purposely causes such suicide by force, duress, or deception. a) Aiding or Soliciting Suicide as an Independent Offense. A person who purposely aids or solicits another to commit suicide is guilty of a felony of the second degree if his conduct causes such suicide or an attempted suicide, and otherwise of a misdemeanor Statutory modification of CL classifications - In some jurisdictions, murder is divided into degrees by statute. A murder will be second degree murder unless it comes under the following circumstances, which would make it first degree murder. nd a. All murders are 2 degree murders unless the prosecution proves any of the following, which st would make the murder 1 degree murder. b. Deliberate and Premeditated Killings: the defendant made the decision to kill in a cool and dispassionate manner and actually reflected on the idea of killing, even if only for a brief period. 1) Deliberate means that the made the decision to kill in a cool and dispassionate manner. 2) Premeditated means that the actually reflected on the idea of killing, if only for a very brief period. 3) In some jurisdictions, forming intent to commit murder is the same as premeditation and deliberation. 4) P&D may be inferred from: a. The type and character of the weapon used, b. The manner in which the weapon was used, c. The nature, extent, and location of the wounds, and d. The accuseds conduct. Degrees of murder First Degree Mens rea requirement 1. Deliberate 2. Premeditated killing 3. Certain types of felony murder (BARKRM) 1. No deliberation and premeditation 2. Certain jurisdictions classify their felony as second-degree murder. d.

c)

Second Degree

c.

Murder in the first degree: murder perpetrated by means of poison, or by lying in wait, or by any other kind of willful, deliberate or premeditated killing, or which shall be committed in the perpetration, or attempted perpetration, of arson, rape, robbery, or burglary. 1) Elements: a. Premeditation means that the killer must have reflected upon and thought about the killing in advance. 1. No particular length of time is required for premeditation. b. Deliberation refers to the quality of the accused thought processes. 2. Refers to killing with a cool head (a careful weighing of the proposed decision). 2) 3 Categories of evidence:

17

Criminal Law Levy II 2011


a. Planning activity b. Manner of killing c. Motive 3) Commonwealth v. Carroll argument in bed rolled over and shot wife 4) People v. Anderson balancing test w/prior acts, prior relationship, & nature of killing st d. Murder in the second degree: all murder not qualifying as 1 degree; intentional killing that is not premeditated and deliberate. Can be because of diminished mental capacity 1) Unintentional Homicide: second degree murder, extreme indifference a. Express malice: intent to kill b. Implied malice: focuses on a defendants awareness of the risk created by his or her behavior People v. Knoller dangerous dogs killed neighbor d) Felony Murder: As the definition of malice aforethought makes clear, a killing (even an accidental one) committed during the course of a felony is murder. The rule provides the any killing committed during the course of a felony is murder. Malice is implied from the intent to commit the underlying felony. a. Common Law: One is guilty of murder if a death results from conduct during the commission or attempted commission of a felony. b. MPC 210.2(1)(b) presumes reckless indifference when a killing occurs during the commission of a specified list of felonies. (same as felony murder rule) 1) Recklessness and indifference presumed if the actor is engaged or is an accomplice in the commission of, or an attempt to commit, or flight after committing or attempting to commit robbery, rape, or deviate sexual intercourse by force or threat of force, arson, burglary, kidnapping or felonious escape. c. What felonies are included? Under the common law, there were only a handful of felonies but the criminal codes of most states have added many more. However, most courts limit the felony murder doctrine to felonies that are inherently dangerous. 1) The prosecution need not show that the killing was deliberate or premeditated. 2) The felonies most commonly listed include arson, robbery, burglary, rape, mayhem, and kidnapping. d. Rationale: added incentive not to participate in felonies, vindicates societies additional calls for retribution when a death occurs e. Rejections: cannot be deterred from committing an accidental act, culpability should be ties to mens rea f. Limitations: 1) Enumeration: Legislatures may list out certain felonies that qualify for felony murder treatment, implying that felonies not listed do not qualify for felony murder treatment. 2) The defendant must be guilty of the underlying felony and the death must have been a foreseeable result of the felony. 3) The felony must be distinct from the killing itself (e.g., commission of aggravated battery that causes a victims death does not qualify as an underlying felony for felony murder liability). 4) The felony must be inherently dangerous to human life (based on the primary element of the offense at issue and factors elevating the offense to a felony.) This usually means that in the abstract the felony cannot be committed without a severe risk to human life. 5) Merger Rule: Courts may hold that the felony murder rule can be applied only where the predicate felony is somewhat independent of the killing. If the underlying felony is the assault or battery by which the victims death is caused, the felony merges into the killing and thus does not retain sufficient independence to be a predicate felony. 6) The death must have been caused before the defendants immediate flight from the felony ended; once the felon has reached a place of temporary safety, subsequent deaths are not felony murder. 7) Agency or Furtherance: Legislatures may limit the felony murder rule to cases in which the or his accomplices (agents) perpetrate the homicide and relieve responsibility if the homicide is the result of someone else.

18

Criminal Law Levy II 2011


Sometimes the rule does not apply when the victim is a co felon. In most jurisdictions, the defendant is not responsible for the death of a co-felon caused by resistance from the felony victim or the police (although the defendant may be responsible for similar deaths of third parties). Provocative Act Doctrine: 1. If the actions of a felon create an atmosphere of malice that provokes a third party into committing the killing, the felons are guilty of murder 2. Technically not a felony murder principle because there is an actual finding of malice created by the co felons provocative behavior 3. Sometimes known as vicarious liability

g.

S.

Sexual Offenses and Rape


a. Under common law, rape is a felony, and was defined as unlawful sexual intercourse without consent by force, fear or deception. a) Common law evidentiary requirements: 1. Victim resisted attack 2. Additional corroborative evidence 3. Reported attack quickly to the police. b) Elements of rape: 1. Sexual intercourse actus reus for the crime. The slightest penetration is sufficient; there need not be completion of sexual act. 2. Unlawful - At common law and under MPC 213.1, a woman must not have been married to the man who committed the act. Today, however, most states have either dropped this requirement where the parties are estranged or separated, or abolish it entirely. 3. Without consent Sex without consent is rape if the defendant was aware or should have been aware that he is acting without the victims consent. 1) If the victim is incapable of consenting, the intercourse is rape. (1) This may be caused by unconsciousness, by the effect of drugs or alcohol substances, or by the victims mental condition. (2) If the victim is so insane or retarded as to be incapable of giving consent, intercourse with her constitutes rape. 4. By force or fear 1) Common law: must prove that intercourse was also accomplished by force, fear or fraud (shown through victims resistence). No set amount of force to constitute rape. 2) Today, most jurisdictions only require either consent or use of force and abandoned the resistance requirement. 3) If intercourse is accomplished by actual force, no question of consent is raised. (1) Commonwealth v. Berkowitz no force used on victim 4) If intercourse is accomplished by placing the victim in fear of great and immediate bodily harm, it constitutes rape. (1) Any consent obtained by such threats is ineffective. (2) Under modern view, the failure of the victim to resist to the utmost does not prevent intercourse from being rape if resistance is prevented by such threats. 5. Or by deception 1) False proclamations of love and commitment or failure to disclose the true reason for wanting to have sex. 2) If the fraudulently persuades the victim that he is her husband, is the intercourse rape? The best answer is no. 3) Other kinds of fraud will not make the intercourse rape. (1) Example: D promises to marry V at a later time and thereby induces V to consent to intercourse. D never intended to marry V. Is D guilty of rape? No. 4) If the victim is fraudulently caused to believe that the act is not sexual intercourse, the act of intercourse constitutes rape medical exam and it is an instrument not a penis. c) Mistake as a defense

19

Criminal Law Levy II 2011


Some courts have held that an honest mistake to consent could constitute a defense to rape because it demonstrated that the defendant did not have the necessary mens rea for the crime. 2. Majority: requires the defendant to have both honest and reasonable belief that the victim had consented. Rape is a second degree felony UNLESS there is serious injury or the victim had not previously allowed sexual encounters with the defendant. Third degree felony if a gross sexual imposition compels to submit when a reasonable person would not, deception. Statutory Rape: a) This is the carnal knowledge of a female who is under the age of consent. 1. Even if the female willingly participated, the offense is nevertheless committed because consent is irrelevant. 2. The age of consent varies from state to state, generally from 16 to 18. b) In most jurisdictions a mistake of fact as to the victims age is IRRELEVANT and cannot be a defense. In some jurisdictions and under the MPC 213.6(1), a reasonable mistake will be allowed if a reasonable person would have also believed the victim was of lawful age to consent. c) Will a s reasonable mistake as to the victims age prevent liability for statutory rape? 1. The best answer is no, because statutory rape is a strict liability crime. 1) Garnett v. State mentally retard 20yr old had sex w/ 14yr old 2. A second best answer, to be used only if no alternative making use of the best position is presented, is that a reasonable mistake as to age will prevent conviction is the reasonably believed the victim was old enough to give an effective consent. drinking in bar 3. People v. Hernandez girl under 18 because mistake of fact is allowed no strict liability Bigamy: a) This is traditionally a strict liability offense that consists of marrying someone while having another living spouse. 1. At common law, a is guilty of bigamy even if she reasonably believes that a purported divorce is valid or that her spouse is dead. Model Penal Code Article 213. Sexual Offenses a) 213.0 Definitions 1. Sexual Intercourse includes intercourse per os and per anum, with some penetration, however slight; emission is not required; 2. Deviate sexual intercourse means sexual intercourse between two human beings who are not husband and wife, and any form of sexual intercourse with an animal. b) 213.1 Rape and Related Offenses 1. Rape. A male who has sexual intercourse with a female not his wife is guilty of rape if: 1) He compels her to submit by force or by threat of imminent death, serious bodily injury, extreme pain or kidnapping, to be inflicted on anyone; or 2) He has substantially impaired her power to appraise or control her conduct other means for the purpose of preventing resistance; or 3) The female is unconscious; or 4) The female is less than 10 years old. 2. Rape is a felony of the second degree unless (i) in the course thereof the actor inflicts serious bodily harm upon anyone, or (ii) the victim was not a voluntary social companion of the actor upon the occasion of the crime and had not previously permitted him sexual liberties, in which case the offense is a felony of the first degree. 3. Gross Sexual Imposition. A male who has sexual intercourse with a female not his wife commits a felony of the third degree if: 1.

b.

c.

d.

e.

20

Criminal Law Levy II 2011


1) 2) 3) he compels her to submit by any threat that would prevent resistance by a woman of ordinary resolution; or he knows that she suffers from a mental disease or defect which renders her incapable of appraising the nature of her conduct; or He knows that she is unaware that a sexual act is being committed upon her or that she submits because she mistakenly supposes that he is her husband.

T.

Causation
a. b. Causation: the tort conception of causation is insufficient to impose criminal responsibility. Instead, a stricter test, requiring a closer connection between the conduct and the resulting harm, may be applied. When a crime is defined to require not merely conduct, but also a specified result of that conduct, the s conduct must be both the cause-in-fact and the proximate cause of the specified result. a) Cause-in-fact: The s conduct must be the cause-in-fact of the result, i.e. the result would not have occurred but for the s conduct. Defendant does not have to be the sole cause 1) Defendant does not have to be the only cause of victims death 1) Commonwealth v. Rementer drunk approached car which hurriedly drove off running over drunk b) Proximate Causation (legal): Problems of proximate causation only arise when the victims death occurs because of the s acts, but in a manner not intended or anticipated by the . 1. The ultimate harm needs to be something that was or should have been reasonably foreseeable as being reasonably related. 1) People v. Kibbe left guy on side of road with no clothes in 0 degrees c) The general rule is that the is responsible for all results that occur as a natural and probable consequence of his conduct, even if he did not anticipate the precise manner in which they would occur. All such results are proximately caused by the s act. The chain of proximate causation is broken only by the intervention of a superseding factor. Common Law Requirement: The death of the victim must occur within one year and one day from the infliction of the injury or wound. a) If it does not occur within this period of time, there can be no prosecution for homicide, even if it can be shown that but for the s actions, the victim would not have not as and when he did. b) The rule has been sharply criticized by the US Supreme Court as an outdated relic of the common law, and most of the states that have recently reviewed the rule have abolished it. Rules of Causation a) An act that hastens an inevitable result is nevertheless a legal cause of that result. 1. Example: A terminates the life support system of B, resulting in Bs death. Can A be held liable for Bs death Yes. Note that society may not wish to condemn such an act of mercy. Nevertheless, for purposes of causation analysis, As act caused Bs death. b) Simultaneous acts by two or more persons may be considered independently sufficiently causes of a single result. 1. Others will say that that the second person accelerated the death and is guilty of murder while the other person is guilty of attempted murder c) A victims pre-existing condition that makes him more susceptible to death does not break the chain of causation. 1. The takes the victim as he finds him. 2. Example: A, with malice aforethought, shoots B in the leg. B bleeds to death before he can receive medical attention because he is a hemophiliac. A is liable for murder despite the fact that a person without hemophilia would not have died from the shooting. Intervening Acts:

c.

d.

e.

21

Criminal Law Levy II 2011


As a general rule, an intervening act will shield the from liability if the act is a mere coincidence or is outside the foreseeable sphere of risk created by the s act. 1. Acts of Nature: A is driving negligently. To avoid As swerving car, B takes an unaccustomed route home. Bs car is struck by lightning, and B dies. Can A be charged with manslaughter? No. The fact that Bs car was struck by lightning was a mere coincidence. 3. Acts by a Third Party: Unless the medical treatment is intentional, or grossly incompetent, bad medical treatment does not break the chain of causation even ordinary negligence 4. Acts by Victim: Ordinarily acts by the victim do not break the chain of causation, even fatal ones, because it is foreseeable that victims will do desperate things to escape from difficult situations, even harm themselves 1) This rule may apply even if the victim acts affirmatively to harm himself. Suppose B, in unbearable pain, commits suicide. The suicide may be found to be a foreseeable consequence of As actions. Stephenson v. State 5. Omissions: third party omissions rarely serve as a superseding intervening cause even if a third party has a duty to rescue and does not the defendant is still guilty 6. Complimentary and concurrent acts: some courts will hold that mutual encouragement in the joint criminal enterprise was sufficient to prove proximate cause. 7. Transferred Intent: Defendant is still guilty however issue arises in whether the defendant is responsible for the harm she intended or the harm she actually caused. 1) MPC: defendant only liable for the harm she intended to cause 2) Common Law: in some jurisdictions the defendant is guilty for harm caused Summary, Analytical Approach a) In analyzing any homicide situation, the following questions must be asked and answered: 1) Did the have any of the states of mind sufficient to constitute malice aforethought? 2. Did the act with intent to kill? 3. Did the act with intent to cause serious bodily harm? 4. Did the act with awareness of high risk that death or serious bodily harm would be caused? 5. Was the death caused in the commission of a felony sufficient to invoke the felony murder rule? b) If the answer to (a) is yes, is there any proof of anything that will, under any applicable statute, raise the homicide to first degree murder? 1) Is there an applicable statute that separates degrees of murder? If so, look to it first to see what qualifies as first and second degree murder. c) If the answer to (a) is yes, is there evidence to reduce the killing to voluntary manslaughter (i.e. adequate provocation)? 2) If the facts suggest that the killing was murder, do those facts also include all four elements of adequate provocation? You must have: 1. a sufficient provocation, 2. a showing that the provocation did in fact stimulate the s killing of the victim, 3. a lack of a cooling off period between the provocation and the killing, 4. the failure of the to have actually cooled off after the provocation. d) If the answer to (a) is no, is there sufficient basis for holding the crime to be involuntary manslaughter (i.e. criminal negligence or misdemeanor manslaughter)? 1) Did the act without malice aforethought but with criminal negligence? 2) Was the death of the victim cause during the commission of an unlawful act that does not bring into play the felony murder rule? a)

f.

22

Criminal Law Levy II 2011


Watch out for a statute that creates the crime of criminally negligent homicide. If such a statute is applicable, a negligent killing will constitute this crime rather than involuntary manslaughter. e) Is there adequate causation between the s acts and the victims death? 1) Did the victim die within one year and one day? 2) Was the s act the factual cause of death? 3) Is there anything to break the chain of proximate causation between the s act and the victims death? Model Penal Code: a) 2.03: Causal Relationship between Conduct and Result; Divergence Between Result Designed or Contemplated and Actual Result or Between Probable and Actual Result. 1) Conduct is the cause of a result when: 1. It is an antecedent but for which the result in question would not have occurred; and 2. The relationship between the conduct and the result satisfies any additional causal requirements imposed by the Code or by the law defining the offense. 2) When purposely or knowingly causing a particular result is an element of an offense, the element is not established if the actual result is not within the purpose or the contemplation of the actor unless: 1. The actual result differs from that designed or contemplated, as the case may be, only in the respect that a different person or a different property is injured or affected or that the injury or harm designed or contemplated would have been more serious or more extensive that that caused; or 2. The actual result involves the same kind or injury or harm as that designed or contemplated and is not too remote or accidental in its occurrence to have a [just] bearing on the actors liability or on the gravity of his offense. 3) When recklessly or negligently causing a particular result is an element of an offense, the element is not established if the actual result is not within the risk of which the actor is aware or, in the case of negligence, of which he should have been aware unless: 1. The actual result differs from the probable result only in the respect that a different person or different property is injured or affected or that the probable injury or harm would have been more serious or more extensive than that caused; or 2. The actual result involves the same kind of injury or harm as the probable result and is not too remote or accidental in its occurrence to have a [just] bearing on the actors liability or on the gravity of his offense. 4) When causing a particular result is a material element of an offense for which absolute liability is imposed by law, the element is not established unless the actual result is a probable consequence of the actors conduct. 3)

g.

T.

Attempt, Solicitation, & Conspiracy


a) The inchoate offenses are solicitation, attempt, & conspiracy. a. They are quite frequently considered felonies. b. An inchoate offense, or anticipatory offense, is committed prior to and in preparation for what may be a more serious offense. An act that bears criminal responsibility before the completion of the targeted offense. c. It is a complete offense in itself, EVEN THOUGH the act to be done MAY NOT have been completed. b) Merger: At common law, under the doctrine of merger, inchoate offenses were regarded as misdemeanors; IF the principal offense was carried out, they were considered felonies. a. The doctrine of merger has been abandoned in many jurisdictions in cases involving a conspiracy. b. HOWEVER, an accused CANNOT be convicted of either attempt or solicitation AND the principal offense.

23

Criminal Law Levy II 2011


c) Attempt: a. A criminal attempt is an act that, ALTHOUGH done with the intention of committing a crime, falls short of completing the crime. Misdemeanor under common law b. Elements 1) An intent to do an act or cause a result which constitutes a crime 2) An act in furtherance of that intent that goes beyond mere preparation (a substantial step) c. A charged with a completed crime MAY BE found guilty of either the completed crime or an attempt to commit the crime AS LONG AS the evidence presented supports such a verdict. 1) The reverse is NOT true. A charged ONLY with an attempt MAY NOT be convicted of the completed crime. d. Intent: 1) The MUST have the intent to perform an act and obtain a result that, if achieved, would constitute a crime. 1. REGARDLESS of the intent for a completed offense, an attempt ALWAYS requires a specific intent. a. For example, attempted murder requires the specific intent to kill another person, EVEN THOUGH the mens rea for murder itself DOES NOT necessarily require a specific intent to kill. 2) A crime defined as the negligent production of a result CANNOT be attempted because IF there were an intent to cause such a result, the appropriate offense would be attempt to intentionally commit the crime rather than attempt to negligently cause the harm. 1. The same is true for recklessness, it CANNOT be attempted. 3) ALTHOUGH a strict liability crime DOES NOT require criminal intent, to attempt a strict liability crime, the MUST act with the intent to bring about the proscribed result. 4) MPC: a defendant who acts with the purpose of causing or with belief that his conduct will cause the prohibitive result satisfies the mens rea standard for attempt. Attempt if the defendant purposely engages in conduct which would constitute the crime if the attendant circumstances were as he believes them to be e. Overt Act: 1) The MUST have committed an act beyond mere preparation for the offense. 1. Several tests have been used to determine whether the act requirement for attempt liability has been satisfied: 2) First step test 1. Rarely employed b/c once it is clear that the defendant has the purpose to commit a crime, anything the defendant does that could lead to the completion of a crime would be a sufficient actus reus to make the defendant guilty of attempt 2. Has been used in poisoning cases 3) Last step test 1. Leave it to the point where only bad luck stopped the crime from happening 2. Has been criticized because it delays law enforcement until long after the defendant has manifested his intent and ability to harm another person 4) Physical Proximity test- Minority jurisdiction 1. Physically near completion of the act 5) Dangerous proximity test 1. How close the defendant came to committing the act 2. More flexible standard measures how much was done and how much was left to do 3. Six factors a. How many steps the defendant has taken b. How much more action is required for the defendant to complete the harmful act c. Why the harm never occurred

24

Criminal Law Levy II 2011


The amount of harm likely to result The seriousness of the prospective harm The appropriateness of law enforcement interference with the defendants acts 6) Unequivocality test 1. Whether the defendants actions, viewed in the abstract, demonstrate an unequivocal intent to commit a crime 2. Often criticized as setting too high a barrier to conviction 7) Indispensable element test 1. Variation of the proximity test 2. If all the indispensible elements have been completed the defendant is guilty of attempt 8) Probable non-desistance test 1. If the defendant goes beyond the point at which someone who wanted to stop would have stopped the defendant has met the actus reus 9) Substantial step approach - MPC Approach and Majority view 1. More than half the courts have adopted this approach 2. 5.01(2) provides that the defendant must take a substantial step strongly corroborative of the actors criminal purpose a. Certain acts that per se satisfy attempts actus reus requirement 3. Still does not provide an exact standard as to when the defendant crosses the line d) Attempt Defenses a. Abandonment: Gives defendants an incentive to abandon their criminal purpose 1) Modern Common Law If defendant voluntarily and completely stops his criminal efforts 2) MPC 5.01(4) renunciation 1. It is an affirmative defense if a. The defendant abandons his effort to commit the crime or prevents it from being committed; and, b. The defendants conduct manifests a complete and voluntary renunciation of his criminal purpose 2. If defendant abandoned his crime because he was scared of getting caught or looking for an easier crime this is not a defense 3. If a has, with the required intent, gone beyond preparation, may she escape liability by abandoning her plans? The general rule is that abandonment is NOT a defense. 1. The MPC approach is that withdrawal WILL be a defense, but ONLY IF: a. It is fully voluntary and NOT made because of the difficulty in completing the crime OR because of an increased risk of apprehension. b. It is a complete abandonment of the plan made under circumstances manifesting a renunciation of criminal purpose NOT JUST a decision to postpone committing it or to find another victim. b. Impossibility: 1) Factual Impossibility (no defense): intends to commit the crime but the facts of the case stop it 1. Look at what he intended to do. 2. It is NO defense to an attempt that it would have been impossible for the to complete her plan i.e. do all those things that she intended to do. This is a factual impossibility. d. e. f.

25

Criminal Law Levy II 2011


Example: A stops B on the street, points a gun at her, and asks her to hand over her money. Unbeknownst to A, B has no money. Is A guilty of attempted robbery? YES. Example: pick pocketing an empty pocket 2) Legal Impossibility (defense): intends to commit the crime but the act is not actually a crime 1. Look at what actually occurred. 2. There used to be hybrid impossibility which was both legal and factual impossibility and it was up to the court to decide whether you should be punished or not but now the ONLY defense is PURE legal impossibility. Therefore, if it is a crime anywhere in the books there is no defense. a. Examples: no law prohibits the defendants behavior chewing gum etc. b. NOTE: Legal impossibility defined in this way would also be a defense under the MPC because the result is not proscribed by law. 3) MPC: Impossibility is not a defense defendant is guilty if he purposely engages in conduct which would constitute the crime if the attendant circumstances were as he believed them to be 4) Note: 1. Mistake of fact = defense 2. Factual impossibility = no defense 3. Mistake of law = no defense 4. Legal impossibility = defense Solicitation merges into the crime committed a. At common law, it was a misdemeanor to solicit another to commit a felony or an act that would breach the peace or obstruct justice. 1) Modern statutes often retain the crime of solicitation, BUT some restrict it to the solicitation of certain serious felonies. b. Elements of Solicitation: 1) Encouraging, requesting, or advising another person to commit a crime 2) With the specific intent that the other person commit the acts constituting that crime c. The offense is complete at the time the solicitation is made. It is NOT necessary that the person solicited agree to commit the crime or do anything in response. 1) If the party solicited committed the crime, the solicitor would be liable for the crime as a party. 2) If the person solicited proceeded far enough to be liable for attempt, the solicitor would be a party for the attempt. 3) If the person solicited refused to commit the crime, the solicitor would still be guilty of solicitation. 4) If the person guilty of solicitation uses a third party to commit the crime but the third party does not know what they are doing the defendant is guilty for the crime d. Solicitation generally is NOT an attempt to commit the crime solicited. This distinction is important in jurisdictions where there is NO crime of solicitation or where the crime of solicitation DOES NOT extend to as many offenses as does the crime of attempt. 1) Differences between attempt and solicitation 1. Attempt requires some act towards the crime solicitation happens much earlier 2. Solicitation requires a third part but attempt can be done alone 3. Attempt generally carries a heavier penalty e. Defenses 1) It is NOT a defense that the solicitation could not have been successful, as where the person solicited was a police undercover agent. 1. The culpability of the solicitor is measured by the circumstances as she believed them to be. 2) Once the solicitation has been made, it is generally NO defense that the solicitor changed her mind or countermanded her advice or urging. 3) There is no first amendment defense for a speech that otherwise constitutes a criminal act 4) If the solicitor could NOT be guilty of the intended crime because of legislative intent to exempt her, she WOULD have a defense. a.

e)

26

Criminal Law Levy II 2011


Example: A minor female could NOT be guilty of solicitation of statutory rape by urging a man to have intercourse with her because she could NOT be guilty of the completed crime. 5) Only possible defense under common law: Renunciation to assert the defense the defendant must 1. Completely and voluntarily renounce her criminal intent; and, 2. Either persuade the solicitated party not to commit the offense or otherwise prevent him from committing the crime a. Have to be prevent forever not just in that moment Accomplice Liability: a. The common law distinguished four types of parties to a felony: 1. Principals in the First Degree: Persons who actually engage in the act or omission that constitutes the criminal offense. 2. Principals in the Second Degree: Persons who aid, command, or encourage the principal and are present at the crime. 3. Accessories Before the Fact: Persons who aid, abet, or encourage the principal, but are NOT present at the crime; AND ------------------------------------------------------------------------------------------------4. Accessories After the Fact: Persons who assist the principal after the crime. b. At common law, the distinction between the parties has a great deal of procedural significance. For example, an accessory COULD NOT be convicted UNLESS the principal had already been convicted, although both could be convicted in a joint trial IF the jury determined the principals guilt first. 1. MOST jurisdictions have abandoned this requirement, and an accessory can now be convicted EVEN IF the principal has evaded apprehension or has been tried and acquitted. c. MOST jurisdictions have abolished the distinctions from common law. Under the modern approach, ALL parties to the crime CAN BE found GUILTY of the criminal offense. 1. A principal is one who, with the requisite mental state, actually engages in the act or omission that causes the criminal result. a. ALSO, anyone who acts through an innocent, irresponsible, or unwilling agent is classified as a principal. b. Example: A gives a poisonous drink to B to give to C. B does so; C drinks it and dies. If B did not know that the drink was poisonous, or if B was mentally ill or under duress, A, not B, is the principal. Note that the principal NEED NOT be present when the harm results. 2. Generally all people involved are subject to the same punishment except for accessory after the fact 3. An accomplice is one who, with the intent that the crime be committed, aids, counsels, or encourages the principal before or during the commission of the crime. 4. An accessory before the fact is one who plans, cooperates, assists, aids, counsels or abets in the preparation of a felony 5. An accessory after the fact is one who receives, relieves, comforts, or assists another knowing that he has committed a felony, in order to help the felon escape arrest, trail, or conviction. a. The crime committed by the principal MUST be a felony, and it MUST be completed at the time the aid is rendered. d. Mens Rea Requirement: i. Purposeful Standard 1. It is not enough that does something that assists with a crime, to be guilty the prosecutor must prove that the defendant spoke with or acted with the purpose to encourage or assist another in the commission of a crime a. State v. Gladstone: a state of opinion is not purpose to encourage or asist 2. Judge Hand: to be guilty as an accomplice, a defendant must not only know that his acts may assist the commission of a crime, but must also have the specific purpose of having the crime succeed. a. MPC: purpose of promoting or facilitating the commission of the crime 1.

f)

27

Criminal Law Levy II 2011


g) Conspiracy does not merge a. At common law, a conspiracy was defined as an agreement between two or more persons to accomplish some unlawful or criminal purpose OR to accomplish a lawful purpose by unlawful means. b. Conspiracy is an inchoate offense, just like attempt and solicitation. a) These are anticipatory crimes, or crimes committed while the s are trying to accomplish other criminal objectives (the end crime). c. Elements: (At common law) a) Agreement between two or more people, b) Intent to enter into an agreement, AND c) Intent to achieve the objective of the agreement. i. Under the traditional definition of conspiracy, the agreement itself was the culpable act. In other words, once you agree, the crime is complete. d) An overt act in furtherance of the agreement i. No required in common law but, a majority of states require an overt act in furtherance of the conspiracy, but mere preparation will USUALLY suffice (UNLIKE attempt, where mere preparation will USUALLY NOT suffice.). 1. This overt act/substantial step/mere preparation MAY BE performed by ANY one of the conspirators, EVEN IF it is unknown to the others. 2. An overt act is any legal or illegal act done by any of the conspirators to set the conspiracy in motion does not take much, a simple call to the bank to see what time they open starts the process for bank robbery Yates v. U.S. ii. Purpose to promote the unlawful act that is the object of the conspiracy iii. Knowledge that the unlawful act is indeed unlawful d. MPC: Conspiracy is a crime of the same grade and degree as the most serious offense which is an object of the conspiracy MPC also merges conspiracy a) Common law is less of an offense than the actual crime but can be charged with both e. Actus Reus requirement of conspiracy a) Act = agreement + overt act b) The agreement may be expressed or implied c) One of the most frequently used arguments is that the conspirators demonstrated their agreement though concerted actions d) It is sufficient that the defendant knows he is agreeing with others to commit a crime he need not know who they are and they do not need to all join at the same time f. Mens Rea requirement a) Requires two i. An intent to agree 1. Defendant intended her actions to signal her decision to join the conspiracy ii. With the purpose to commit a crime 1. Knowledge alone is insufficient to convict someone of conspiracy. Have to prove that the defendant had a stake in the venture or had the purpose to facilitate the crime 2. People v Lauria seller must know buyers intended illegal use and purposely intend to promote the act b) The defendant need only know of those attendant circumstances the defendant would need to know for the substantive crime. Example crime to beat someone did not need to know that the person was a federal officer c) It is VERY difficult to separate the intent to agree from the act of agreement. i. Hence, MOST courts DO NOT even try. The intent to agree CAN BE inferred from conduct.

28

Criminal Law Levy II 2011


ii. The MUST intend to achieve the objective of the conspiracy. The intent MUST BE established AS TO EACH, individual, . d) A person who acts with the intent to facilitate a conspiracy may thereby become a member of the conspiracy. HOWEVER, intent CANNOT be inferred from mere knowledge. i. Therefore, a merchant who sells a good in the ordinary course of business that he knows will be used to further a conspiracy DOES NOT thereby join the conspiracy. 1. Special circumstances, such as a stake in the venture or a sale of illegal items, MUST occur. e) Conspiracy is a specific intent crime. Therefore, a conspiracy to commit a strict liability crime, for which intent is NOT required, requires intent. i. Example: A and B agree on a scheme to persuade C, a 17-year-old girl to have intercourse with them. They believe she is 21, but this would not be a defense to the completed crime of statutory rape. Can they be convicted of conspiracy to commit statutory rape? No, because conspiracy requires knowledge of the victims age even though the completed crime does not. Co-conspirator Liability (the pinkerton rule) a) A conspirator is responsible for all acts of her co-conspirators during the course of and in furtherance of the conspiracy, even if the conspirator is unaware that these acts are happening Pinkerton v US b) Broader than accomplice liability and does not apply retroactively only guilty of crimes while in c) It is sufficient that the crime was reasonably foreseeable and a natural consequence of the conspiracy d) MPC has rejected the pinkerton rule a co-conspirator is only guilty of the substantive crime if there is evidence of accomplice liability Hearsay Exception: the statement of one co conspirator is deemed to be a vicarious admission by an agent of the other partners to the conspiracy Merger: Under the old rule, if the conspirators were successful and completed their crime, the crime of conspiracy merged into the completed crime. a) While the members of the agreement COULD BE convicted of the completed crime, they COULD NOT BE convicted of the conspiracy. b) This is NO LONGER the law in MOST jurisdictions. NOW, the majority rule is that if conspirators are successful, they CAN BE convicted of both the criminal conspiracy AND the crime they committed pursuant to the conspiracy. Accomplice Liability: One conspirator MAY, by virtue of his participation in the scheme, meet the requirements of aiding and abetting the commission of crimes by his co-conspirators and therefore be liable for those crimes as an accomplice. a) EVEN IF the conspirator DID NOT have the sufficient mental state for accomplice liability, a separate doctrine provides that each conspirator MAY BE LIABLE for the crimes of all the other conspirators IF two requirements are met: i. The crimes were committed in furtherance of the objectives of the conspiracy. ii. The crimes were probable and natural consequence of the conspiracy: foreseeable. b) This doctrine applies ONLY IF the conspirator HAS NOT made a legally effective withdrawal from the conspiracy before the commission of the crime by the co-conspirator. Scope of the agreement single or multiple conspiracies a) Defendants often argue that there were multiple small conspiracies and that the crime committed was a part of one that the defendant was not involved in b) Wheel conspiracy i. If the separate spokes of the wheel have a vested interest in the success of one anothers illegal conduct, there is a single wheel conspiracy and each individual member is responsible for the crimes of every other member of the conspiracy

g.

h. i.

j.

k.

29

Criminal Law Levy II 2011


ii. However, if the only connection between the spokes is that they know the same middleman then there are multiple small conspiracies and the spokes of the wheel are not liable for each others acts iii. Easiest way to link them all together is to prove that the defendants relied on the success of the other spokes did not happen in Kotteakos v. U.S. c) Chain conspiracy i. Conspirators participate in a single conspiracy by performing different roles along a single distribution line ii. Classic example is distribution of narcotics 1. Manufacturerwholesalerdistributor U.S. v. Bruno d) Multiple objectives to one conspiracy i. If a conspiracy has a goal to commit multiple crimes it is still often considered one conspiracy e) Rules regarding parties to conspiracy i. Can argue that the agreement was not between two or more persons that qualified as conspirators to a conspiracy ii. Gebardi Rule 1. A person who is protected by the law that the defendants are conspiring to violate cannot be a valid party to the conspiracy iii. Wharton Rule 1. If it is impossible to commit the substantive offense without cooperative action, the preliminary agreement between the parties to commit the offense is not an indictable conspiracy 2. Prostitution needs two people to happen therefore in order to be charged with conspiracy there must be a third person involved iv. Bilateral Rule 1. A conspiracy requires at least two guilty minds that is, at least two persons who can actually be prosecuted for conspiracy plurality requirement a. Most common = feigned conspirator i. Police officer is the second party 2. Under the MPC unilateral approach, conspiracy is established by showing: i. That the agreed with another, ii. To commit the crime, b. Regardless of whether the other person shared in the commitment Duration of conspiracy and abandonment/withdrawal defense a) A conspiracy begins the minute two or more persons agree to commit a crime and it lasts until it has been abandoned or its objectives have been achieved i. Abandoned when none of the conspirators are engaging in any action to further the conspiracy actions - Act of concealment is not considered part of the conspiracy ii. MPC: the statute of limitations begins to run once the defendant has to notify the other members of the conspiracy or notified the authorities. Defendant could also thwart the crime and get off b) Impossibility is NOT a defense to conspiracy. EVEN IF it was impossible to achieve the ultimate objective of the conspiracy, the s CAN BE found guilty of the conspiracy itself. i. Similarly, the governments defeat of the conspiracys ultimate objective DOES NOT automatically terminate the conspiracy. United States v. Jimenez-Recio

l.

30

Criminal Law Levy II 2011


The general rule is that withdrawal from a conspiracy is NOT a defense to a charge of conspiracy, because the conspiracy is complete AS SOON AS the agreement is made AND an overt act is committed. i. However, the MPC 5.03(7)(c) recognizes voluntary withdrawal as a defense IF the thwarts the success of the conspiracy (e.g. by informing the police). ii. A person MAY limit his liability for subsequent acts of the other members of the conspiracy, including the target crime for which conspiracy was formed, if he withdraws. 1. To withdraw, he MUST perform an affirmative act that notifies ALL the members of the conspiracy, and such notice MUST BE given in time for them to have the opportunity to abandon their plans. a. Judge this on a level of reasonableness. Just like within attempt, withdrawal MUST BE complete and voluntary. b. NOTE if he has also provided material assistance SO AS to be liable as an accomplice, he MUST attempt to neutralize the assistance. c. People v. Sisselman - point of defense is to thwart the required mental state m. Criminal enterprises and RICO a) Racketeer Influenced and Corrupt Organizations i. Elements: U.S. v. Horak 1. Using income derived from a pattern of racketeering activity to acquire an interest in an enterprise 2. Acquiring or maintaining an interest in an enterprise through a pattern of racketeering activity 3. Conducting the affairs of an enterprise through a pattern of racketeering activity 4. Conspiring to commit any of these offenses ii. The defendant must be associated with an enterprise which is defined as a legal or illegal organization engaging in ongoing activities 1. If the defendants engage in racketeering two or more crimes of a certain type through or affecting the enterprise, the defendants are guilty of a RICO violation iii. No need to fit RICO defendants into a wheel or chain b) Two differences between RICO and other conspiracies are i. Activity that would be viewed as multiple conspiracies under traditional law may be charged together under RICO as a pattern of racketeering activity; and, ii. Evidence of other conspirators crimes that are unrelated to a specific racketeering activity are admissible against a RICO defendant n. Felony murder and co-conspirator liability a) Whenever you see multiple defendants and a murder three things to come to mind conspiracy, accomplice liability and felony murder. c)

T.

Justification & Excuses


1. Justification and Excuses a. Justification Renders a nominal violation of the law lawful and exempt from criminal sanctions. b. Excuses the act is wrong, but it is understood why the actor is relieved from responsibility. Self Defense: (Justification) a. Non-deadly Force: As a general rule, an individual who is without fault may use such force as reasonably necessary to protect herself from the imminent use of unlawful force upon herself.

2.

31

Criminal Law Levy II 2011


There is no duty to retreat before using non-deadly force, even if retreat would result in no further harm to with party. b. Deadly Force: A person may use deadly force in self defense if he reasonably believes that said force is necessary to protect himself reasonableness is determined by what the defendants circumstances dictate to a reasonable person People v. Goetz a) Elements: 1. An honest and reasonable fear of death or great bodily harm a. Honest is up to the jury b. MPC 3.04 = defendant may use deadly force if fears becoming victim of felony crime (rape, robbery etc.) c. Reasonable when society agrees w/ defendant reasonable defendant 2. From an imminent and unlawful threat a. No threat of imminent death therefore no defense = State v. Norman 3. Proportional response to that threat and a. No excessive force is permitted 4. Defendant was not the initial aggressor a. One cannot create his own necessity to use self-defense b. MPC 3.04 can invoke self defense if original threat of force was less than attempted force by other party 5. Minority of jurisdictions require duty to retreat a. Applied only when defendant knows he can retreat w/ complete safety b. No such duty in own house castle rule People v. Tomlins c. MPC 3.04(2)(b)(ii)(1) does not require when i. Where the attack occurs in the victims house. ii. Where the attack occurs while the victim is making a lawful arrest, and iii. Where the assailant is in the process of robbing the victim b) Imperfect Self-Defense 1. When not all the elements of self defense are met = partial defense 2. Honest but unreasonable fear can change murder to manslaughter 3. MPC 3.09 mistake as to the need of force is a defense except for offenses that only require recklessness or negligence Original Aggressor: Generally, one who begins a fight has no right to use force in her own defense during that fight. But an aggressor can regain her right to self-defense in two ways: a) Withdrawal: An aggressor who, in good faith, effectively removes herself from the fight, and communicates to the other person her desire to remove herself, regains her right to use selfdefense. b) Sudden Escalation: If the victim of the initial aggression suddenly escalates a minor fight into one involving deadly force and does so without giving the aggressor the chance to withdraw, the aggressor may use force in her own defense. Defense of Others: a) There are two issues in determining whether a person who has used force to defend another person is criminally liable for her acts: rd 1. Majority: defendant may use force in defense of 3 person if he reasonably believes rd such force is necessary to defend 3 person from imminent unlawful attack rd 2. Minority: defendant stands in 3 persons shoes and no defense unless person under attack would have had the right to use self defense 1.

c.

d.

32

Criminal Law Levy II 2011


Relationship: The majority rule is that there does not have to be some special relationship with the person is criminally liable for her acts. Defense of a Dwelling: a. Non-deadly Force: A person is justified in the use of non-deadly force in defense of her dwelling when, and to the extent that, she reasonably believes that such conduct is necessary to prevent or terminate anothers unlawful entry into or attack upon her dwelling. b. Deadly Force: One is generally justified in the use of deadly force in two situations: a) Use of deadly force is justifiable where the entry was made or attempted in a riotous, violent, or tumultuous manner and the person reasonably believes that the use of force is necessary to prevent a personal attack upon herself or another in the dwelling. b) Use of deadly force is justifiable where the person reasonably believes that such force is necessary to prevent the entry into the dwelling by a person who intends to commit a felony in the dwelling. Defense of Property: a. Non-deadly force: This force may be used to defend property in ones possession from unlawful interference. a) In the case of real property, this means entry or trespass b) In the case of personal property, this means removal or damage. c) The need to use force must reasonably appear imminent. Thus, force may not be used if a request to desist or refrain from the activity would suffice. b. Deadly Force: Defense of property alone can never justify the use of deadly force. a) A person may use deadly force in defense of property only in conjunction with another privileged use of force. (i.e. self-defense, defense of others.). Duress (Excuse) a. Allowed because the defendant has acted without a fair opportunity to exercise free will and therefore not deserving of punishment a) Different from necessity b/c not necessarily the lesser of two evils and another person forced act b. Common law elements a) A threat of death or serious bodily harm 1. Economic harm never qualifies b) Imminently posed c) Against the defendant or a close friend or relative d) That creates such fear that an ordinary person would yield 1. Ordinary reasonable man, not a battered woman e) The defendant did not put themselves in the situation f) The defendant is not seeking to raise the defense to a homicide 1. Homicide situations: in a few jurisdictions one may argue imperfect duress if committed murder under duress c. Additional situations a) Mistaken threats: if the defendant/reasonable person reasonably believed that the threat was real b) Brainwashing: some jurisdictions brainwashing is seen as a defense b/c it deprives the defendant of free will very difficult to do d. MPC 2.09 a) Threat of unlawful force, no separate imminency requirement, against any person, person of reasonable firmness in defendants situation would yield 3.

3.

4.

5.

33

Criminal Law Levy II 2011


Limitations of defense: defendant did not recklessly put himself in duress situation (if charged with crime of negligence, no duress defense if defendant negligently put himself in duress situation + allows for defense in homicide cases e. Note: An act committed under duress is termed excusable rather than justifiable. 1. The subtle distinction stems from the fact that criminal acts performed under duress are condoned by society rather than encouraged. f. Most jurisdictions also impose another element on the defense of duress that the had no reasonable opportunity to escape the threatened harm. a) Sometimes, a fourth element is also required that the submit to the proper authorities after attaining a position of safety. Necessity a. Conduct otherwise criminal is justifiable if, as a result of pressure from natural forces, the reasonably believed that the conduct was necessary to avoid some harm to society that would exceed the harm caused by the conduct. b. Choice of lesser evils defense if the defendant is faced with two evils and chooses the lesser of the two that person is not deserving of punishment a) Common law elements US v Maxwell (necessity does not arise from defendants choice of a preferred course of action - there was a legal alternative) 1. Defendant faces a choice of evils a. Economic necessity is insufficient 2. There are no apparent legal alternatives a. If there is a lawful alternative the defendant must pick it 3. There is an imminent harm a. If the threat is in the future the defendant had time to find another alternative 4. Defendant chooses the lesser harm a. Loss of life is a greater evil than loss of property b. Too difficult to decide which life is more valuable risks fewer lives to save more 5. Additionally the courts impose the following restrictions a. Defendant did not create the necessity i. MPC 3.02 a defendant who creates her own necessity des not lose the right to assert a defense for intentional crimes can be prosecuted for negligent or reckless crimes b. There has been no legislative decision to prohibit the necessity defense in that situation i. If there is no necessity regardless if choice is lesser of two evils b) MPC elements 3.02 1. Harm avoided is greater than harm done 2. There is no specific prohibition to the use of a choice of evils defense for this offense 3. There is no clear legislative purpose to exclude the choice of evils defense in the defendants situation a. MPC is broader than the common law definition in three ways i. No imminency requirement ii. No absolute prohibition on self-created necessity iii. Necessity is available in homicide prosecutions 1.

6.

34

Criminal Law Levy II 2011


c) The test is objective, a good faith belief in the necessity of ones conduct is insufficient. 1. Causing the death of another person to protect is never justified. 2. The defense of necessity is not available if the defendant is at fault in creating a situation which requires that she choose between two evils. 3. Example: Throwing cargo overboard during a violent storm, if necessary to save the lives of the crew and other people on board a ship, would not constitute criminal damage to property. On the other hand, throwing some members of the crew overboard to save the cargo would never be justifiable. d) To distinguish, duress involves a human threat and necessity involves pressure from physical or natural human forces. 1. Example: A points a gun at B and threatens to kill B if she does not break into Cs house and steal food. B may raise the defense of duress. If, however, B is a starving victim of a plane crash in a desolate area and commits the same act, she has the defense of necessity. e) Prison Escapes: intolerable conditions v escape. Defendant must surrender himself immediately upon reaching a place of safety Elements from People v. Lovercamp used in State v. Reese 1. 2 approaches to using the elements pre requisites: no element no defense OR Factors: take the elements as adding weight to the defendants testimony Competency to stand trial: a. Focuses on the defendants mental state at the time of trial a) If competency cannot be reached the trial is usually dropped and the defendant is committed b. A defendant is mentally competent to stand trial if she has sufficient ability to a) Consult with her attorney b) To rationally understand the proceedings Diminished Capacity: a. Controversial defense allows a defendant to argue that she was not able to form the intent for the crime charged because she did not have the mental capacity b. After the twinkie defense many jurisdictions have banned the diminished capacity defense Insanity: a. Rationale for insanity defense a) A person who does not know what they are doing cannot be deterred from their actions b) Can commit to a mental facility w/o adding stigma of criminal status c) Incapable of reason, just like punishing an animal just wrong d) Insane are deprived of their free will and therefore never exercised it committing a crime b. Disease of defect of the mind a) A defendant must suffer from a mental disease or defect of the mind b) Any abnormal condition of the mind that substantially affects mental or emotional processes and substantially impairs behavior controls. c) Includes following factors: 1. Does the condition have clear symptoms, do the medical and scientific communities support the recognition of this condition as a medical defense, is this a condition defendants are likely to bring up themselves, is this condition easily feigned. How frequently will this condition be invoked for an insanity defense, and are there policy reasons to exclude or include this condition as a disease?

7.

8.

9.

35

Criminal Law Levy II 2011


c. The various formulations differ significantly on what effects a mental illness must have had to entitle the to an acquittal. a) Remember that insanity is a legal term as opposed to a psychiatric one. b) It is also a generic term, encompassing many possible mental abnormalities, all of which have one thing in common they are recognized by law as dictating certain consequences. 1. Usually, the cause of a s mental illness or insanity is irrelevant in determining the legal consequences. Formulations of the Insanity Defense: 1. MNaghten Rule (strictest of all tests): The traditional rule provides that a is presumed sane but is entitled to an acquittal if the proof establishes that: a. At the time of commission of the offense b. The defendant was laboring under a defect of disease of the mind. c. And, the defendant did not know d. The nature and quality of his acts; or, e. That his acts were wrong i. Wrong is morally or legally. Morally to societies norms ii. Know the act was wrong does NOT mean know it was against the law f. Example: thinks he is shopping up a watermelon when in fact it is a head 2. Deific command exception: most jurisdictions now allow an insanity defense when the defendant, due to a mental disease or defect, believes that God or a higher being ordered her to commit the crime 0 may know it is against societal norms but that God has overridden the norms Key is THE JURY HAS TO BELIEVE IT 3. Under next 3 tests even if defendant knew the act was wrong can still use as a defense 4. Irresistible Impulse Test (rejected by a lot of jurisdictions mental disease implied): Under this test, a is entitled to an acquittal if the proof establishes that: a. Because of a mental illness, the was unable to: i. Control his actions or ii. Conform his conduct to the law. b. Contrary to what the name of this test might imply, this inability need not come upon the suddenly. A number of jurisdictions apply both MNaghten and the IIT. Thus, a person would be entitled to acquittal if he meets either test. 5. Durham Product Rule (not widely accepted): Under this test, a is entitled to an acquittal if the proof establishes that his unlawful act was the product of the mental disease or defect. a. A crime is a product of the disease if it would not have been committed but for the disease. In this way, the Durham test is broader than either the MNaghten or IIT. It was intended primarily to give psychiatrists greater liberty to testify concerning the s mental condition. 6. MPC Test 4.01: Under this test, the is entitled to an acquittal if the proof shows that: a. He suffered from a mental disease or defect. b. And as a result lacked substantial capacity to either: iii. Appreciate the criminality or wrongfulness of his conduct (does not need to know it is wrong appreciate means capacity to appreciate the nature of the act), or iv. Conform his conduct to the requirements of law. c. This test combines both the MNaghten and the IIT by allowing for the impairment of both cognitive and volitional capacity. This test is rapidly becoming the most popular formulation.

d.

36

Criminal Law Levy II 2011


Federal Test: Under this test, a is entitled to an acquittal if proof shows that at the time of the commission of the acts constituting the offense, the , as a result of a mental disease or defect, was unable to appreciate the nature and the quality of the wrongfulness of his acts. e. Notes on Insanity: 1. Many formulations expressly exclude the psychopathic criminal, the person who repeatedly commits crimes without experiencing any guilt. a. This is usually accomplished by defining mental illness so as to exclude any abnormally evidenced only by repeated antisocial conduct. Sociopathic and psychopathic are synonymous. 2. All s are presumed sane. The insanity issue is not raised until the comes forward with some evidence tending to show that he was insane under the applicable test. 3. In most jurisdictions, acquittal by reason of insanity puts into operation a procedure which the acquitted may be committed to a mental institution until cured. a. The confinement of an insanity acquittee in a mental hospital, based solely on the trial courts finding of insanity, may last until he has regained his sanity or is no longer dangerous. This may be longer than the maximum period of incarceration carried by his offense. 4. Some states recognize the defense of diminished capacity. Under this defense, the may assert that as a result of a mental defect short of insanity, he did not have the particular mental state required for the crime charged. b. Most states recognizing the defense limit it to specific intent crimes. 10. Status Defenses: a. Infancy: Children under a certain age are presumed incapable of forming the intent required for a criminal offense, but different jurisdictions choose different ages a) some hold it as a full defense whereas others hold it as a rebuttal presumption b) Common Law: 1. A child under the age of 7 did not have the cognitive capacity to form the mes rea for any crime prosecution could not rebut this 2. A minor between the ages of 7 14 was presumed incapable of committing a crime but the prosecution could rebut this presumption by demonstrating that the youth was capable of understanding the nature and consequences 3. A youth over 14 had no infancy defense c) MPC 410: does not set forth an age at which a defendant is not responsible for their acts d) If a youth is 14 and it is a serious crime they may be charged in an adult court instead of juvenile 11. Entrapment: allows a defendant to be excused because the government unfairly induced that defendant to commit a crime full defense a. To successfully invoke the entrapment defense, the defendant must first demonstrate that he was induced by a government official or informant to commit a crime. a defendant cannot be entrapped by a third party not working for the government. Secondly he must show that he was either not predisposition to commit the crime, or if a law abiding citizen would have been induced to commit the crime depends on the jurisdiction b. Two standards to tell if entrapped: a) Subjective (federal predisposition) test: if the defendant was predispositioned to commit the crime and law enforcement agents only offered him the opportunity to do so, there is no entrapment defense. 1. Jury may consider prior criminal acts etc. 7.

37

Criminal Law Levy II 2011


Defendant can remove the case if the government engaged in outrageous misconduct dismissals under this theory are rare and occur only when the government constructs the crime from beginning to end b) Objective (government induced) test: focuses on the nature of the governments conduct. Whether a law abiding citizen would have been induced to commit the crime c) MPC Approach 2.13: leaves the determination to the court not the jury 2.

38

Вам также может понравиться